Discussion Forum

AMC 10 Mock Exam Discussion For 9/20: Question 21

 
 
Picture of John Lensmire
AMC 10 Mock Exam Discussion For 9/20: Question 21
by John Lensmire - Wednesday, September 20, 2023, 1:56 PM
 

AMC 10 Mock Exam Question 21

Two non-decreasing sequences of nonnegative integers have different first terms. Each sequence has the property that each term beginning with the third is the sum of the previous two terms, and the seventh term of each sequence is $N$. What is the smallest possible value of $N$?

$\displaystyle \textbf{(A) } 55 \qquad \textbf{(B) } 89 \qquad \textbf{(C) } 104 \qquad \textbf{(D) } 144 \qquad \textbf{(E) } 273$

Submit your answer and solution and explanation below! Solutions will be accepted for 48 hours until 9/22 at 2pm Pacific Time. (There's still time for yesterday's problem too: click here.)

Top solutions for all the Mock Exam questions will be collected and shared as part of a full 25 Question Mock AMC 10 Exam.

Note: The question above is a past AMC problem. Solutions submitted must be written by students. Copied solutions will be disqualified.